Last visit was: 24 Apr 2024, 03:53 It is currently 24 Apr 2024, 03:53

Close
GMAT Club Daily Prep
Thank you for using the timer - this advanced tool can estimate your performance and suggest more practice questions. We have subscribed you to Daily Prep Questions via email.

Customized
for You

we will pick new questions that match your level based on your Timer History

Track
Your Progress

every week, we’ll send you an estimated GMAT score based on your performance

Practice
Pays

we will pick new questions that match your level based on your Timer History
Not interested in getting valuable practice questions and articles delivered to your email? No problem, unsubscribe here.
Close
Request Expert Reply
Confirm Cancel
SORT BY:
Date
Math Expert
Joined: 02 Sep 2009
Posts: 92901
Own Kudos [?]: 618698 [156]
Given Kudos: 81586
Send PM
Most Helpful Reply
GMAT Club Legend
GMAT Club Legend
Joined: 08 Jul 2010
Status:GMAT/GRE Tutor l Admission Consultant l On-Demand Course creator
Posts: 5957
Own Kudos [?]: 13386 [44]
Given Kudos: 124
Location: India
GMAT: QUANT+DI EXPERT
Schools: IIM (A) ISB '24
GMAT 1: 750 Q51 V41
WE:Education (Education)
Send PM
User avatar
Manager
Manager
Joined: 15 Feb 2012
Status:Perspiring
Posts: 71
Own Kudos [?]: 373 [26]
Given Kudos: 216
Concentration: Marketing, Strategy
GPA: 3.6
WE:Engineering (Computer Software)
Send PM
RC & DI Moderator
Joined: 02 Aug 2009
Status:Math and DI Expert
Posts: 11161
Own Kudos [?]: 31880 [19]
Given Kudos: 290
Send PM
Re: A list of numbers has six positive integers. Three of those integers a [#permalink]
9
Kudos
10
Bookmarks
Expert Reply
Bunuel wrote:
A list of numbers has six positive integers. Three of those integers are known: 4, 5 and 24 and three of those are unknown: x, y and z. The three unknowns are known to be distinct. It is also known that the mean of the list is 10 and the median lies between 7 and 8 (exclusive).

Which of the following CANNOT be the value of any one of the unknowns?

(A) 13

(B) 12

(C) 11

(D) 10

(E) 5


Hi all those looking for a short method..

points to NOTE..
1) the median of 6 integers will be the centre value of two MIDDLE integers, here it will be 3rd and 4th in ascending/descending order.
2) when you are looking for centre of two integers, the centre value will be either an integer or a number with .5 in decimals. Here it is given between 7 and 8, the value is 7.5
3) sum of two of the three unknowns is 7.5*2=15


Let's work further..
Total of unknowns=10*6-(24+5+4)=27..
The third unknown =27-15=12..

for other two totalling 15, the lower one cannot be less than 5, as the two lowest are 4 and 5..
So values can be 5 and 10,... or 6 and 9... or 7 and 8...
So A and C are not possible...
Bunuel , we may have to change the median from ' between 7 and 8(exclusive) to from 7 to 8(exclusive)..

So now the Q says that the median could be 7 also,
total of two is 2*7=14 and third unknown becomes 27-14=13....
And other two values can be 5 and 9...... Or 6 and 8.....

Now only 11 is not possible..
Ans C
General Discussion
avatar
Intern
Intern
Joined: 04 Nov 2013
Posts: 26
Own Kudos [?]: 11 [3]
Given Kudos: 10
Concentration: Finance, Strategy
GPA: 4
Send PM
Re: A list of numbers has six positive integers. Three of those integers a [#permalink]
3
Kudos
Bunuel wrote:
A list of numbers has six positive integers. Three of those integers are known: 4, 5 and 24 and three of those are unknown: x, y and z. The three unknowns are known to be distinct. It is also known that the mean of the list is 10 and the median lies between 7 and 8 (exclusive). Which of the following CANNOT be the value of any one of the unknowns?

(A) 13
(B) 12
(C) 11
(D) 10
(E) 5



If the mean is 10, the sum is 60. The three known numbers add to 33.
If the median is between 7 and 8, that means two digits were averaged to the median. (7<(x+y)/2<8 is the equation form).
x+y must be 15 (between 14 and 16). The other number is 12 (to equal 27).

I chose to solve using the answer choices. 10 and 5 add up to 15, so certainly both cannot be the answer.

For 11 to be the answer, the other number is 4.

4,4,5,11,12,27 is the new set. The median is 8. The question said 7 and 8 exclusive. Therefore, 11 is the answer.
C
GMAT Club Legend
GMAT Club Legend
Joined: 08 Jul 2010
Status:GMAT/GRE Tutor l Admission Consultant l On-Demand Course creator
Posts: 5957
Own Kudos [?]: 13386 [3]
Given Kudos: 124
Location: India
GMAT: QUANT+DI EXPERT
Schools: IIM (A) ISB '24
GMAT 1: 750 Q51 V41
WE:Education (Education)
Send PM
Re: A list of numbers has six positive integers. Three of those integers a [#permalink]
2
Kudos
1
Bookmarks
Expert Reply
Bunuel wrote:
A list of numbers has six positive integers. Three of those integers are known: 4, 5 and 24 and three of those are unknown: x, y and z. The three unknowns are known to be distinct. It is also known that the mean of the list is 10 and the median lies between 7 and 8 (exclusive). Which of the following CANNOT be the value of any one of the unknowns?

(A) 13
(B) 12
(C) 11
(D) 10
(E) 5

Kudos for a correct solution.


Set of terms is {4, 5, x, y, z, 24}

Since x, y and z are distinct so let's assume Let, x < y < z

Mean = 10 i.e. Sum = Mean*no. of terms = 10*6 = 60

i.e. Sum of {4, 5, 24, x, y, z} = 60
i.e. x+y+z = 27

median = Average of two middle terms (for even no. of terms in set) = between 7 and 8 which can only be 7.5
if x+y = 7.5*2 = 15
i.e. z = 27 - 15 = 12

Case 1: x = 5 and y = 10 and z = 12
Case 2: x = 6 and y = 9 and z = 12
Case 3: x = 7 and y = 8 and z = 12

OR if z = 13 then x+y = 14
i.e. x and y can be 4 and 10 respectively and the set becomes {4, 4, 5, 10, 13, 24} i.e. Mean = 7.5

So x, y and z can be anything but 11

Answer: Option C
Math Expert
Joined: 02 Sep 2009
Posts: 92901
Own Kudos [?]: 618698 [15]
Given Kudos: 81586
Send PM
Re: A list of numbers has six positive integers. Three of those integers a [#permalink]
8
Kudos
7
Bookmarks
Expert Reply
Bunuel wrote:
A list of numbers has six positive integers. Three of those integers are known: 4, 5 and 24 and three of those are unknown: x, y and z. The three unknowns are known to be distinct. It is also known that the mean of the list is 10 and the median lies between 7 and 8 (exclusive). Which of the following CANNOT be the value of any one of the unknowns?

(A) 13
(B) 12
(C) 11
(D) 10
(E) 5

Kudos for a correct solution.


VERITAS PREP OFFICIAL SOLUTION:

The question gives us concrete information about mean – it is 10 – but not about median – it is between 7 and 8 (exclusive). What can we say about median from this? That it cannot be 7 or 8 but anything in between. But we know that the list has all integers. When we have even number of integers, we know that the median is the average of the middle two numbers – when all are placed in increasing order. So can the average of the two middle numbers be, say, 7.1? Which two positive integers can average to give 7.1? None! Note that if the average of two integers is a decimal, the decimal must be (some number).5 such as 7.5 or 9.5 or 22.5 etc. This happens in case one number is odd and the other is even. In all other cases, the average would be an integer.

Since the median is given to be between 7 and 8, the median of the list of the six positive integers must be 7.5 only.

Now we know that the mean = 10 and median = 7.5

Method 1: Algebra/Logic

Let’s try to solve the question algebraically/logically first.

There are 6 elements in the list. The average of the list is 10 which means the sum of all 6 elements = 6*10 = 60

4 + 5 + 24 + x + y + z = 60

x + y + z = 27

Median of the list = 7.5

So sum of third and fourth elements must be 7.5 * 2 = 15

There are two cases possible:

Case 1: Two of the three integers x, y and z could be the third and the fourth numbers. In that case, since already 4 and 5 are less than 7.5, one of the unknown number would be less than 7.5 (the third number) and the other two would be greater than 7.5.

The sum of the third and fourth elements of the list is 15 so

15 + z = 27

z = 12

So, two numbers whose sum is 15 such that one is less than 7.5 and the other greater than 7.5 could be

5 and 10

6 and 9

7 and 8

x, y and z could take values 5, 6, 7, 8, 9, 10 and 12.

Case 2: The known 5 could be the third number in which case one of the unknown numbers is less than 5 and two of the unknown numbers would be more than 7.5.

If the third number is 5, the fourth number has to be 10 to get a median of 7.5. Hence, 10 must be one of the unknown numbers.

The sum of the other two unknown numbers would be 27 – 10 = 17.

One of them must be less than 5 and the other greater than 10. So possible options are

4 and 13

3 and 14

2 and 15

1 and 16

x, y and z could take various values but none of them could be 11

Answer (C)

Method 2: Process of Elimination

Let’s now try to look at the process of elimination here and see if we can find an easier way.

The three unknowns need to add up to 10*6 – 4 – 5 – 24 = 27.

Two of the given options are 5 and 10. They have a median of 7.5 so lets assume that two of the unknown numbers are 5 and 10 (5 can be one of the unknowns since we are not given that all six integers need to be distinct). If two unknowns make up third and fourth numbers in the list and have a median of 7.5, their sum would be 15 and the third unknown will be 12 (to get the mean of 10). This case (5, 10, 12) satisfies all conditions so options (B), (D) and (E) are out of play.

Now we are left with two options 13 and 11. Check any one of them and you will know which one is not possible. Let’s check 13.

From the given options, any number greater than 7.5 must be either the fourth number or the fifth number. 13 cannot be the fourth number since the third number would need to be 2 in that case to get median 7.5. But we have 4 and 5 more than 2 so it cannot be the third number. So 13 must be the fifth number of the list. We saw in the case above that if two unknowns are third and fourth numbers then the fifth number HAS TO BE 12. So the already present 5 must be the third number and the fourth number must be 10. In that case, the leftover unknown would be 4 (to get a sum of 27). So the three unknowns would be 4, 10 and 13. This satisfies all conditions and is possible. Hence answer must be (C). 11 will not be possible.

Let’s see what would have happened had you picked 11 to try out. If 11 were the fourth number, to get a median of 7.5, we would need 4 as the third number. That is not possible since we already have a 5 given. So 11 must have been the fifth number. This would mean that the already present 5 and one unknown 10 would make the median of 7.5. So the third unknown in this case would be 6 (to get a sum of 27). But 6 would be the third number and the median in this case would be (6 + 10)/2 = 8. So one of the numbers cannot be 11.

Answer (C)
Director
Director
Joined: 17 Dec 2012
Posts: 589
Own Kudos [?]: 1519 [0]
Given Kudos: 20
Location: India
Send PM
A list of numbers has six positive integers. Three of those integers a [#permalink]
Expert Reply
The sum of 3 numbers is 33
The sum of remaining numbers is 27 since mean is 10
The fourth number is 10 since the third number is 5 and the median is between 7 and 8
The sum of the remaining 2 numbers has to be 17. If we try to fit to the choices we can see they can be 13 and 4 or 12 and 5
11 does not figure in the list.
So C.
Manager
Manager
Joined: 18 Dec 2017
Posts: 159
Own Kudos [?]: 18 [1]
Given Kudos: 13
Send PM
Re: A list of numbers has six positive integers. Three of those integers a [#permalink]
1
Bookmarks
GMATinsight wrote:
Bunuel wrote:
A list of numbers has six positive integers. Three of those integers are known: 4, 5 and 24 and three of those are unknown: x, y and z. The three unknowns are known to be distinct. It is also known that the mean of the list is 10 and the median lies between 7 and 8 (exclusive).

Which of the following CANNOT be the value of any one of the unknowns?

(A) 13

(B) 12

(C) 11

(D) 10

(E) 5


Terms are {4, 5, 24, x, y, z}
Mean = 10 i.e. Sum = 10*6 = 60
i.e.4+5+24+x+y+z = 60
i.e. x+y+z = 27

Media = between 7 and 8
But since number of terms in the set is even so median is the average of two middle terms which can only be 7.5
Hence, Median = 7.5
Media 7.5 is possible when two of the terms in the sets are {7,8} or {6,9} or {5,10} or {4,11} or {3,12} or {2,13} or {1,14}

Lets check options
(A) 13 {x, y, z} may be {2, 13, 12} hence Possible

(B) 12 {x, y, z} may be {5, 10, 12} hence Possible

(C) 11 {x, y, z}hence NOT Possible as the pair needed is {4, 11} but 4 can't be third term ever as sum of other two of x, y, z is 16 which can't be each smaller than 4. CORRECT ANSWER

(D) 10

(E) 5

Answer: Option C


How could A give result of set with median of 7.5? It is 2, 4, 5, 12, 13, 24 then the median becomes 8.5 is it not?
UNC Kenan Flagler Moderator
Joined: 18 Jul 2015
Posts: 238
Own Kudos [?]: 247 [1]
Given Kudos: 120
GMAT 1: 530 Q43 V20
WE:Analyst (Consumer Products)
Send PM
Re: A list of numbers has six positive integers. Three of those integers a [#permalink]
1
Kudos
Bunuel wrote:
A list of numbers has six positive integers. Three of those integers are known: 4, 5 and 24 and three of those are unknown: x, y and z. The three unknowns are known to be distinct. It is also known that the mean of the list is 10 and the median lies between 7 and 8 (exclusive).

Which of the following CANNOT be the value of any one of the unknowns?

(A) 13
(B) 12
(C) 11
(D) 10
(E) 5


1. The set would look like \(x = {4, 5, 24, x, y, z}\) not in any specific order

2. We are given that the mean of set \(x\) is \(10\) i.e. \(\frac{4 + 5 + 24 + x + y + z}{6} = \frac{33 + x + y + z}{6} = x + y + z = 27\)

3. We are also given that median is between \(7\) and \(8\) i.e. \(7.5\). Hence, the sum of the middle \(2\) terms will be \(15\)

4. The first combination that I can think of is \({4, 5, 5, 10, 12, 24}\). The mean of this set is \(10\) and median is \(7.5\). B, D and E can be eliminated

5. The second combination that I can think of is \({4, 4, 5, 10, 13, 24}\). We can eliminate A

Ans. C
Tutor
Joined: 16 Oct 2010
Posts: 14817
Own Kudos [?]: 64890 [1]
Given Kudos: 426
Location: Pune, India
Send PM
A list of numbers has six positive integers. Three of those integers a [#permalink]
1
Kudos
Expert Reply
Bunuel wrote:
A list of numbers has six positive integers. Three of those integers are known: 4, 5 and 24 and three of those are unknown: x, y and z. The three unknowns are known to be distinct. It is also known that the mean of the list is 10 and the median lies between 7 and 8 (exclusive).

Which of the following CANNOT be the value of any one of the unknowns?

(A) 13
(B) 12
(C) 11
(D) 10
(E) 5


Known integers: 4, 5 and 24
Median is the avg of the middle two integers. The avg of two integers will be Integer/2 and hence will either be an integer or integer.5
Since median lies between 7 and 8, it must be 7.5. So the middle two numbers could be (7, 8) or (6, 9) or (5, 10). The middle two numbers cannot be (4, 11) because if 4 is the 3rd number, the 4th number will be 5. No other values of middle two numbers are possible.

Mean = 10 so sum of all numbers is 60.
If the middle two numbers are (7, 8) or (6, 9), we are left with only one unknown and it must be 12 ( = 60 - 4 -5 - 24 - 15)
If the middle two numbers are (5, 10), the list could be 4, 5, 5, 10, 24 in which case only one unknown is left which must be 12.
or the list could be 4, 5, 10, 24 with two unknowns such that one of them is smaller than 5. If another unknown is 4, the last unknown becomes 13.

So our unknowns can take the values of 5, 10, 12 and 13, but not 11.

Answer (C)

Now try this question on mean, median and range: https://anaprep.com/sets-statistics-mea ... -question/

Originally posted by KarishmaB on 24 Aug 2022, 23:59.
Last edited by KarishmaB on 18 Nov 2023, 01:20, edited 1 time in total.
Senior Manager
Senior Manager
Joined: 21 Nov 2021
Posts: 437
Own Kudos [?]: 209 [0]
Given Kudos: 343
Send PM
A list of numbers has six positive integers. Three of those integers a [#permalink]
Bunuel wrote:
A list of numbers has six positive integers. Three of those integers are known: 4, 5 and 24 and three of those are unknown: x, y and z. The three unknowns are known to be distinct. It is also known that the mean of the list is 10 and the median lies between 7 and 8 (exclusive).

Which of the following CANNOT be the value of any one of the unknowns?

(A) 13
(B) 12
(C) 11
(D) 10
(E) 5


The 3 unknowns add up to 27.

If one of the numbers is less than 5, then 5 becomes the highest of the 3 and is matched with the next highest number and divided by 2 to be between 7&8.

That means the next highest number could only be 10.

Choosing 4 as the lower number and 10 being the required other number, means that

13 could be one unknown, along with 10

Choosing 3 or lower isn't necessary since it only increases from 13 and those numbers aren't among the choices.

Now, if instead all 3 numbers are >5, trying 6 as the next highest number means that only 9 can be the next, leaving

27-9-6= 12 as a possibility

Finally, choosing 5 as a number means again that 10 has to be another, leaving 27-5-10=12.

So by elimination, only 11 can be the correct choice.

Posted from my mobile device
Intern
Intern
Joined: 23 Feb 2023
Posts: 7
Own Kudos [?]: 1 [1]
Given Kudos: 26
Send PM
A list of numbers has six positive integers. Three of those integers a [#permalink]
1
Kudos
Question about median could be a really tricky one... I wanted to visualize it.

Condition 1) Median is 7.5 (numbers are all integers)
Condition 2) x + y + z = 27. So, two numbers should be added up to 15 and remaining number should be 12.

------3---4---5---6---7-(median)-8---9---10---11---12---13-----------------------------24

1) Draw a number line & Eliminate answer choices quickly ----> 12 is already eliminated(above), 5 & 10 can easily eliminated.
2) 11 vs. 13 ---> Number line tells me to eliminate 11. (because we already have "5", so can't match 4-11 )
But to verify, try 13. ---> 13, 10, 4 could be matched because we already have "5" !!!

Therefore answer is C.
Attachments

54274.jpg
54274.jpg [ 176.68 KiB | Viewed 1361 times ]

User avatar
Intern
Intern
Joined: 29 Feb 2024
Posts: 1
Own Kudos [?]: 1 [1]
Given Kudos: 8
Send PM
Re: A list of numbers has six positive integers. Three of those integers a [#permalink]
GMATinsight wrote:
Bunuel wrote:
A list of numbers has six positive integers. Three of those integers are known: 4, 5 and 24 and three of those are unknown: x, y and z. The three unknowns are known to be distinct. It is also known that the mean of the list is 10 and the median lies between 7 and 8 (exclusive).

Which of the following CANNOT be the value of any one of the unknowns?

(A) 13

(B) 12

(C) 11

(D) 10

(E) 5


Terms are {4, 5, 24, x, y, z}
Mean = 10 i.e. Sum = 10*6 = 60
i.e.4+5+24+x+y+z = 60
i.e. x+y+z = 27

Media = between 7 and 8
But since number of terms in the set is even so median is the average of two middle terms which can only be 7.5
Hence, Median = 7.5
Media 7.5 is possible when two of the terms in the sets are {7,8} or {6,9} or {5,10} or {4,11} or {3,12} or {2,13} or {1,14}

Lets check options
(A) 13 {x, y, z} may be {2, 13, 12} hence Possible

(B) 12 {x, y, z} may be {5, 10, 12} hence Possible

(C) 11 {x, y, z}hence NOT Possible as the pair needed is {4, 11} but 4 can't be third term ever as sum of other two of x, y, z is 16 which can't be each smaller than 4. CORRECT ANSWER

(D) 10

(E) 5

Answer: Option C

­If we solve via this way, then how are you eliminating Option A? for {2,13,12} the number in ascending order would look like {2,4,5,12,13,24}
Medium here will be 5 + 12 = 17/2 = 8.5(out of scope)
 
Manager
Manager
Joined: 10 Apr 2023
Posts: 51
Own Kudos [?]: 18 [0]
Given Kudos: 58
Location: India
Send PM
Re: A list of numbers has six positive integers. Three of those integers a [#permalink]
­What does  "the median lies between 7 and 8 (exclusive)" mean?

is it that median has to lie between 7 and 8 ?
Math Expert
Joined: 02 Sep 2009
Posts: 92901
Own Kudos [?]: 618698 [0]
Given Kudos: 81586
Send PM
Re: A list of numbers has six positive integers. Three of those integers a [#permalink]
Expert Reply
 
Rohan271 wrote:
­What does  "the median lies between 7 and 8 (exclusive)" mean?

is it that median has to lie between 7 and 8 ?

­
Yes, ­ "the median lies between 7 and 8 (exclusive)" means that the median lies between 7 and 8 (exclusive).
GMAT Club Bot
Re: A list of numbers has six positive integers. Three of those integers a [#permalink]
Moderators:
Math Expert
92901 posts
Senior Moderator - Masters Forum
3137 posts

Powered by phpBB © phpBB Group | Emoji artwork provided by EmojiOne